Latest Banking jobs   »   Reasoning Quiz for Indian Bank PO...

Reasoning Quiz for Indian Bank PO Mains : 15th October 2018


Dear Aspirants,

Reasoning Quiz for Indian Bank PO Mains : 15th October 2018

Reasoning Questions for Indian Bank PO Mains

Reasoning Ability is an onerous section. With the increasing complexity of questions, it becomes hard for one to give it the cold shoulder. The only way to make the grade in this particular section in the forthcoming banking exams like Indian Bank PO Mains is to practice continuously with all your heart and soul. And, to let you practice with the best of the latest pattern questions, here is the Adda247 Reasoning Quiz based on the exact same pattern of questions that are being asked in the exams.


Directions (1-5): Study the information carefully and answer the questions given below. 

Six members of a family A, B, C, D, E and F are sitting in a row facing north. There are three generation and no member in the third generation is married. D’s wife sits exactly between D and A. Three persons sit between E and her brother. C is grandmother of E. B has two children. D is father-in-law of A. B sits immediate left to her daughter. A sit next to his son. C sits immediate right to her husband and none of them sits at any end.

Q1. Who among the following is the brother of E?

A
F
C
B
none of these
Solution:

Reasoning Quiz for Indian Bank PO Mains : 15th October 2018 |_3.1



Q2. How many married couples are there in the family?

Two
One
Three
Four
None of these
Solution:

Reasoning Quiz for Indian Bank PO Mains : 15th October 2018 |_4.1



Q3. How is the person sitting 2nd left to A related to A?

father-in-law
brother-in-law
wife
son-in-law
none of these
Solution:

Reasoning Quiz for Indian Bank PO Mains : 15th October 2018 |_4.1



Q4. How many persons are sitting between B and A?

Two
One
Three
Four
None of these
Solution:

Reasoning Quiz for Indian Bank PO Mains : 15th October 2018 |_4.1



Q5. How is D related to F?

grandfather
grandmother
grandson
grand daughter
none of these
Solution:

Reasoning Quiz for Indian Bank PO Mains : 15th October 2018 |_4.1



Directions (6-10): In these questions, relationship between different elements is shown in the statements. These statements are followed by two conclusions. 

If A*B means A is not greater than B 
A%B means A is not greater than or equal to B 
A&B means A is not smaller than B 
A#B means A is not smaller than or equal to B 
A@B means A is not greater than or smaller than B 

Q6. Statements: A%C#Q@S, J#C#B 
       Conclusions: I. S % J 
                              II. S % B

If only conclusion I follows.
If only conclusion II follows.
If either conclusion I or II follows.
If neither conclusion I nor II follows.
If both conclusions I and II follow.
Solution:

I. S % J (true)
II. S % B (false)

Q7. Statements: M#B, H &M %F %A @Q 
       Conclusions: I. B %A  
                              II. B % H

If only conclusion I follows.
If only conclusion II follows.
If either conclusion I or II follows.
If neither conclusion I nor II follows.
If both conclusions I and II follow.
Solution:

I. B %A(true)
II. B % H (true)

Q8. Statements: A # Q #M # R@ S, Q@Z#N 
       Conclusions: I. N % R 
                              II. N & R

If only conclusion I follows.
If only conclusion II follows.
If either conclusion I or II follows.
If neither conclusion I nor II follows.
If both conclusions I and II follow.
Solution:

I. N % R(false)
II. N & R(false)

Q9. Statements: P % B # Q, X%B & A 
       Conclusions: I. P # X 
                              II. A * P

If only conclusion I follows.
If only conclusion II follows.
If either conclusion I or II follows.
If neither conclusion I nor II follows.
If both conclusions I and II follow.
Solution:

I. P # X (false)
II. A * P(false)

Q10. Statements: A & J % Y @ Z # M # R 
         Conclusions: I. A # Y 
                                II. R % Y

If only conclusion I follows.
If only conclusion II follows.
If either conclusion I or II follows.
If neither conclusion I nor II follows.
If both conclusions I and II follow.
Solution:

I. A # Y (false)
II. R % Y(true)

Directions (11-15): Each of the questions below consists of a question and two statements numbered I and II given below it. You have to decide whether the data provided in the statement are sufficient to answer the question. Read both the statements and Given answer: 

Q11. What is the code for ‘Banking’? 
(I) ‘exam like paper’ is written as ‘Q2K L2V D2O’ in that code language. 
(II) ‘events section important’ is written as ‘3%T 2@F 3%J’ in the code language.

If the data in statement I alone are sufficient to answer the question, while the data in statement II alone are not sufficient to answer the question.
If the data in statement II alone are sufficient to answer the question, while the data in statement I alone are not sufficient to answer the question.
If the data either in statement I alone or in statement II alone are sufficient to answer the question.
If the data even in both statements I and II together are not sufficient to answer the question.
If the data in both statement I and II together are necessary to answer the question.
Solution:

From I- Code for banking is – F2Y
From II- Code for banking- 2%H

Q12. Who among R, S, K, P and M is exactly in the middle when they are arranged in ascending order of their heights? 
I. P is taller than S but shorter than M. 
II. K is taller than S and M but shorter than R.

If the data in statement I alone are sufficient to answer the question, while the data in statement II alone are not sufficient to answer the question.
If the data in statement II alone are sufficient to answer the question, while the data in statement I alone are not sufficient to answer the question.
If the data either in statement I alone or in statement II alone are sufficient to answer the question.
If the data even in both statements I and II together are not sufficient to answer the question.
If the data in both statement I and II together are necessary to answer the question.
Solution:

From I, we have: M > P> S.
From II, we have: K > S, K > M, R > K.
Combining the above two,
we have: R>K>M>P>S
Clearly, M is in the middle.

Q13. How is Rita related to Nita? 
I. Prem has only two kids, Rita and Nita. Prem is the mother-in-law of Queen, who is sister-in-law of Nita. 
II. Roma is the sister-in-law of Rita, and the daughter-in-law of Soni, who has only two kids Rita and Nita.

If the data in statement I alone are sufficient to answer the question, while the data in statement II alone are not sufficient to answer the question.
If the data in statement II alone are sufficient to answer the question, while the data in statement I alone are not sufficient to answer the question.
If the data either in statement I alone or in statement II alone are sufficient to answer the question.
If the data even in both statements I and II together are not sufficient to answer the question.
If the data in both statement I and II together are necessary to answer the question.
Solution:

From I, we conclude that Prem is the mother of Rita and Nita, while Queen is the daughter-in-law of Prem and sister-in-law of Nita. Thus, Queen is Rita's wife and hence, Rita is Nita's brother.
From II, we conclude that Rita and Nita are the children of Soni. Also, Roma is the daughter-in-law of Soni and sister-in-law of Rita. So, Roma is Nita's wife and thus, Nita is Rita's brother. Hence, Rita is either brother or sister of Nita

Q14.Eight people viz. G, H, I, J, K, L, M and N lives in a Building on different floors from top to bottom(such as ground floor numbered as 1 and top is numbered as 8) where I lives on floor number 6. Who among the following lives on 4th floor? 
(I)Only one person lives between L and M. J lives above I. There is a gap of three floors between J and L and both of them lives on odd number of floor. 
(II)Three persons live between K and H. N lives just above H who lives on even numbered floor.

If the data in statement I alone are sufficient to answer the question, while the data in statement II alone are not sufficient to answer the question.
If the data in statement II alone are sufficient to answer the question, while the data in statement I alone are not sufficient to answer the question.
If the data either in statement I alone or in statement II alone are sufficient to answer the question.
If the data even in both statements I and II together are not sufficient to answer the question.
If the data in both statement I and II together are necessary to answer the question.
Solution:

Reasoning Quiz for Indian Bank PO Mains : 15th October 2018 |_8.1



Q15.Seven people viz. A, D, F, L, M, Q, R are sitting in a row some of them are facing north while some of them are facing south direction. R sits at an extreme end of the row. F is not facing south direction and does not sit at any end. D sits third to the left of F. Who sits to the immediate right of F? 
(I)M sits third to right of R.Q sits second to the left of M.D is not an immediate neighbor of Q. (II)A sits third to the left of Q. F is an immediate neighbor of M.F sits second to the right of L, who faces south.

If the data in statement I alone are sufficient to answer the question, while the data in statement II alone are not sufficient to answer the question.
If the data in statement II alone are sufficient to answer the question, while the data in statement I alone are not sufficient to answer the question.
If the data either in statement I alone or in statement II alone are sufficient to answer the question.
If the data even in both statements I and II together are not sufficient to answer the question.
If the data in both statement I and II together are necessary to answer the question.
Solution:

Reasoning Quiz for Indian Bank PO Mains : 15th October 2018 |_9.1

               


Print Friendly and PDF

Leave a comment

Your email address will not be published. Required fields are marked *